Pytanie:
Dlaczego niektóre obroty są niestabilne? (Równania Eulera)
John Doe
2018-04-06 02:08:55 UTC
view on stackexchange narkive permalink

Mamy następujące równania Eulera dla wirującego ciała

$$ I_1 \ dot \ omega_1 + \ omega_2 \ omega_3 (I_3-I_2) = 0 \\ I_2 \ dot \ omega_2 + \ omega_1 \ omega_3 (I_1-I_3) = 0 \\ I_3 \ dot \ omega_3 + \ omega_2 \ omega_1 (I_2-I_1) = 0 $$ Gdzie $ I_i $ to momenty bezwładności wokół osi $ x_i $, a $ \ omega_i $ to prędkość kątowa wokół tej osi.

Można wykazać (*), że jeśli $ I_1>I_2>I_3 $, to obiekty o prędkości kątowej bardzo bliskiej $ \ vec \ omega = (0,1,0) $ są niestabilne. Dlaczego tak się dzieje i jak mogę to sobie wyobrazić?

Próbowałem to sobie wyobrazić za pomocą piłki, ale zdałem sobie sprawę, że prawdopodobnie nie jest to dobry sposób na jej wizualizację, ponieważ piłka jest kulisto symetryczna, więc momenty bezwładności nie są wyraźne. Czy jest jakaś wizualizacja lub animacja, która pozwoliłaby mi zobaczyć tę rotację i prawdopodobnie zrozumieć, dlaczego jest niestabilna?


(*) W odpowiedzi na komentarz @ SRS:

Nie jestem pewien co do odniesień, ale wiem, jak to zrobić: Niech $ \ omega_1 = \ eta_1, \ omega_3 = \ eta_3 $ gdzie $ \ eta $ to małe zaburzenie i przypuśćmy, że $ \ omega_2 = 1 + \ eta_2 $. Następnie równania Eulera stają się $$ I_1 \ dot \ eta_1 = (I_2-I_3) \ eta_3 + O (\ eta ^ 2) \ tag1 $$$$ I_2 \ dot \ eta_2 = O (\ eta ^ 2) \ tag2 $ $$$ I_3 \ dot \ eta_3 = (I_1-I_2) \ eta_1 + O (\ eta ^ 2) \ tag3 $$ Rozróżnij $ (1) $ i sub w $ (3) $ na wynikowe wyrażenie $$ \ ddot \ eta_1 = \ frac {(I_2-I_3) (I_1-I_2)} {I_3I_1} \ eta_1 $$ Jeśli $ I_1>I_2>I_3 $, to stała po prawej stronie jest dodatnia, więc rozwiązanie tego równania jest wykładnicze ( gdyby było to jakiekolwiek inne zamówienie, rozwiązaniem byłoby $ \ sin / \ cos $). Dlatego jest niestabilny.


Edit:

Aby wyjaśnić, opublikowałem to pytanie, aby zobaczyć inne, bardziej wizualne sposoby zrozumienia tego efektu, zamiast rozwiązywania równań, jak to zrobiłem powyżej, i aby zobaczyć, jak ten efekt działa w prawdziwym życiu. Więc nie sądzę, że jest to duplikat innych pytań, ponieważ nie mają odpowiedzi, które pasują do tego.

1. https://www.youtube.com/watch?v=PY7fRozbrtk 2. https://www.youtube.com/watch?v=-Si6iRL5Fj8 3, https://www.youtube.com/watch?v=4dqCQqI-Gis (z prawdziwą rakietą tenisową).Zobacz https://en.wikipedia.org/wiki/Tennis_racket_theorem, aby zapoznać się z pracą analityczną.
Właśnie to pokazałeś haha.JEŚLI chcesz to przetestować, najlepszym sposobem jest, jak mówisz, wybranie obiektu z $ I_1 \ neq I_2 \ neq I_3 \ neq I_1 $, a dobre 1 to pudełko na chusteczki lub dowolny pryzmat.
@FGSUZ Tak, wiem jak to matematycznie pokazać, moje pytanie dotyczy tego jak to wizualizować, jak na przykład linki podane przez ZeroTheHero
Możesz to również zademonstrować samodzielnie za pomocą buta.
Możliwe duplikaty: https://physics.stackexchange.com/q/67957/2451, https://physics.stackexchange.com/q/17504/2451, https://physics.stackexchange.com/q/34364/2451i linki w nich zawarte.
@Qmechanic - Zgadzam się, że jest to duplikat tych starszych pytań, ale odpowiedzi tutaj bardziej mi się podobają niż jakiekolwiek inne.Być może te starsze pytania powinny zostać zamknięte jako duplikat tego.
Możliwa kopia [Stabilność obrotu pryzmatu prostokątnego] (https://physics.stackexchange.com/questions/67957/stability-of-rotation-of-a-rectangular-prism)
@Qmechanic Widziałem już jedno z tych pytań, kiedy szukałem, ale innych nie znalazłem.Niezależnie od tego, nie sądzę, że jest to duplikat, ponieważ poprosiłem konkretnie o bardziej wizualny sposób zrozumienia tego, a nie tylko pokazanie, że małe zaburzenie prowadzi do wykładniczego rozwiązania.
Inną dobrą wizualizacją jest rozważenie obracania długiego pręta wokół osi bliskiej, ale nie tej samej, co jego oś środkowa.Kiedy będziesz obracać się dokładnie wokół tej osi środkowej, będzie stabilny.Ale jeśli jesteś trochę oddalony, siły obrotowe odciągną pręt dalej od tej osi.
Trzy odpowiedzi:
Michael Seifert
2018-04-06 03:14:06 UTC
view on stackexchange narkive permalink

Jest jeszcze inny fajny sposób spojrzenia na to matematycznie. Nietrudno wykazać, że w ramie korpusu zachowały się dwie wielkości: kwadrat wektora pędu $$ L ^ 2 = L_1 ^ 2 + L_2 ^ 2 + L_3 ^ 2 $$ i obrotowa energia kinetyczna, która okazuje się być $$ T = \ frac {1} {2} \ left (\ frac {L_1 ^ 2} {I_1} + \ frac {L_2 ^ 2} {I_2} + \ frac {L_3 ^ 2} {I_3} \ right). $$ (Zwróć uwagę, że moment pędu $ \ vec {L} $ sam nie jest zachowany w ramie korpusu; ale tak się składa, że ​​jego kwadrat jest stały).

Następnie możemy zadać pytanie: Jakie są dozwolone wartości $ \ vec {L} $ dla podanych wartości $ L ^ 2 $ i $ T $? Łatwo zauważyć, że ograniczenie $ L ^ 2 $ oznacza, że ​​$ \ vec {L} $ musi leżeć na powierzchni kuli; i prawie tak łatwo jest zobaczyć, że ograniczenie $ T $ oznacza, że ​​$ \ vec {L} $ musi również leżeć na powierzchni danej elipsoidy, z głównymi osiami $ \ sqrt {2TI_1} > \ sqrt {2T I_2} > \ sqrt {2T I_3} $. Zatem dozwolone wartości $ \ vec {L} $ muszą leżeć na przecięciu kuli i elipsoidy. Jeśli utrzymamy $ L ^ 2 $ ustalone i wygenerujemy kilka tych krzywych dla różnych wartości $ T $, wyglądają one następująco:

enter image description here

Zauważ, że dla danej wartości $ L ^ 2 $, obiekt będzie miał najwyższą możliwą energię kinetyczną podczas obracania się wokół osi z najniższym momentem bezwładności i odwrotnie.

Załóżmy zatem, że obiekt obraca się wokół osi swojego największego momentu bezwładności. Jeśli zaburzymy ten obiekt tak, że nieznacznie zmienimy jego energię (zakładając na potrzeby argumentacji, że $ L ^ 2 $ pozostaje stałe), to zobaczymy, że wektor $ \ vec {L} $ będzie teraz leżał na stosunkowo małej krzywej w pobliżu jego pierwotnej lokalizacji. Podobnie, jeśli obiekt obraca się wokół swojej osi o najniższej bezwładności, $ \ vec {L} $ pozostanie stosunkowo blisko swojej pierwotnej wartości, gdy zostanie zakłócony.

Sytuacja jest jednak znacznie inna, gdy obiekt obraca się początkowo wokół osi pośredniej (trzeci czerwony punkt na powyższym schemacie, na „przedniej stronie” kuli. Kontury lekko zakłóconego $ T $ w pobliżu tegonie nie pozostawać w pobliżu osi pośredniej; wędrują po całej kuli. Nic więc nie powstrzymuje $ \ vec {L} $ przed wędrowaniem po całej tej kuli, jeśli nieco odsuniemy obiekt od obracaniawokół tej osi; co oznacza, że obiekt obracający się wokół swojej osi pośredniej jest niestabilny.

Nigdy wcześniej tego nie widziałem.Wciąż uważam, że diagram jest dość abstrakcyjny, ale po obejrzeniu filmów o drążku w kształcie litery T staje się on nieco jaśniejszy - krzywe podążają za większymi liniami przypominającymi elipsy dla niestabilnych oscylacji.Dzięki!:)
Piękna odpowiedź.
Te odpowiedzi - twoje i ZeroTheHero - są po prostu piękne.Nigdy wcześniej nie myślałem o tym w tych terminach: po prostu byłem szczęśliwy, wiedząc, że stabilna linearyzacja układu o punkcie równowagi oznacza, że można znaleźć kulę o niezerowym promieniu, w której można przekształcić problem jako naprawioną skurczowąproblem.Ale - takie piękno też z perspektywy globalnej!
Hej Michael, bawiłem się, próbując napisać program do stworzenia takiej fabuły.Udało mi się zdobyć taki, który wygląda jak fabuła ZeroTheHero, ale nie mogę wymyślić, jak narysować twoją figurę.Czy obliczyłeś ogólną postać równania żółtych krzywych, a następnie wykreśliłeś je?Jeśli tak, czy pamiętasz, czym one były?
@JohnDoe: W przypadku tej wersji wykresu faktycznie dokładnie opracowałem kształt krzywych;nie zmieszczą się one jednak w tym polu komentarza.:-) Skutecznie jednak wyrażasz $ L_1, L_2, L_3 $ we współrzędnych sferycznych, zapisujesz obie powierzchnie w postaci współrzędnych sferycznych, ustawiasz je na równe sobie, używasz tożsamości $ \ sin ^ 2 \ theta = (1- \ cos ^ 2 \ theta) $, a następnie oblicz $ \ cos ^ 2 \ theta $ jako funkcję $ \ phi $.Jeśli jednak używasz Mathematica, łatwiejszym sposobem jest użycie opcji `MeshFunctions`, z energią kinetyczną jako` MeshFunction`.
ZeroTheHero
2018-04-06 06:41:38 UTC
view on stackexchange narkive permalink

Istnieje alternatywa dla metody @MichaelSeifert, która wykorzystuje moment pędu i momenty bezwładności: zajmuje się wektorem $ \ vec \ omega $ bezpośrednio, ponieważ interesuje nas ewolucja tego wektora.

Można wyrazić energię kinetyczną i długość do kwadratu $ \ vec L $ jako \ begin {align} T& = \ frac {1} {2} \ left (I_1 \ omega_1 ^ 2 + I_2 \ omega_2 ^ 2 + I_3 \ omega_3 ^ 2 \ right) \\ \ vec L \ cdot \ vec L& = I_1 ^ 2 \ omega_1 ^ 2 + I_2 ^ 2 \ omega_2 ^ 2 + I_3 ^ 2 \ omega_3 ^ 2 \ end {align} Te dwa równania definiują różne elipsoidy w $ \ omega $ -space. Ponieważ $ \ vec \ omega $ ma trzy składniki, ale tylko dwa ograniczenia, ograniczenia nie wystarczają do całkowitego określenia $ \ vec \ omega $. Raczej ewolucja $ \ vec \ omega $ musi leżeć na przecięciu linii tych dwóch elips, co ilustruje czarna linia na rysunku. (Zielona elipsa jest stała $ \ vec L \ cdot \ vec L $, a żółta stała $ T $.)

enter image description here

To tutaj przecięcie ma z grubsza kształt spłaszczonego banana, z dłuższym bokiem w kierunku $ \ omega_2 $.

Tak więc, podczas ewolucji, wierzchołek $ \ vec \ omega $ może poruszać się wzdłuż tego przecięcia, jednocześnie utrzymując stałą energię i $ \ vec L \ cdot \ vec L $. Rysunek ilustruje przypadek, w którym $$ I_1 = 4, \ quad I_2 = 3/2, \ quad I_3 = 1 $$ z $ T = 5/2 $ i $ \ vert \ vec L \ vert = 2,45 $.

Na rysunku widać, że jeśli zaczniemy od przecięcia na górze, ewolucja $ \ omega_3 $ jest dość ograniczona (i nie może zmienić znaku), że ewolucja $ \ omega_1 $ jest również dość ograniczone, ale składnik $ \ omega_2 $ jest znacznie większy (w zasadzie cała długość spłaszczonego banana). Stąd jakościowo mówiąc , obrót wokół środkowej osi jest niestabilny , ponieważ $ \ omega_2 $ nie musi pozostawać blisko swojej wartości początkowej.

(Ten sam argument działa dla składników $ L_k = \ omega_k I_k $ w figurze MichaelaSeiferta, z tym wyjątkiem, że w jego przypadku $ L_1 $ nie zmieni znaku, ale $ L_3 $ może zmienić znak w pobliżu hiperbolicznego punktu stałego.)

I to też jest piękna odpowiedź.Nawiasem mówiąc, kiedy już się tego nauczysz i wypróbujesz w książce wokół jej środkowej osi i zobaczysz, jak się przewraca, nigdy tego nie zapomnisz.Najlepszą książką, z którą można to zrobić, jest Mechanika klasyczna Goldsteina
Te odpowiedzi - twoje i Michaela - są po prostu piękne.Nigdy wcześniej nie myślałem o tym w tych terminach: po prostu byłem szczęśliwy, wiedząc, że stabilna linearyzacja układu o punkcie równowagi oznacza, że można znaleźć kulę o niezerowym promieniu, w której można przekształcić problem jako naprawioną skurczowąproblem.Ale - takie piękno też z perspektywy globalnej!
W rzeczywistości jest to bardziej tradycyjny sposób, w którym to robi, iw tym przypadku krzywe przecięcia nazywane są [polhodes] (https://en.wikipedia.org/wiki/Polhode) (jedno z moich ulubionych słów dotyczących fizyki).po prostu łatwiej będzie sobie wyobrazić, że jedna z powierzchni ograniczonych jest sferą.
@MichaelSeifert Dziś dowiedziałem się jeszcze jednej rzeczy.Dzięki.
user3294068
2018-04-06 23:20:45 UTC
view on stackexchange narkive permalink

Mój profesor fizyki pomógł nam to sobie wyobrazić za pomocą rakiety tenisowej

Obracanie się z osią wzdłuż rączki jest stabilne.

Kręcenie się w płaszczyźnie rakiety jest stabilne.John McEnroe często rzucał rakietami w ten sposób.

Obracanie się w drugą stronę, tak jak wymachiwanie rakietą, jest niestabilne.Niezależnie od tego, jak ostrożnie odwracasz rakietę, rakieta obraca się również w drugą stronę, zanim wróci do Twojej dłoni.

Jeśli nie masz dostępu do rakiety do gry: https://www.youtube.com/watch?v=4dqCQqI-Gis

Dzięki!Dopóki nie zadałem tego pytania, nie zdawałem sobie sprawy, że czasami nazywa się to twierdzeniem o rakietach tenisowych.Świadomość tego znacznie ułatwiłaby wizualizację tego w prawdziwym życiu


To pytanie i odpowiedź zostało automatycznie przetłumaczone z języka angielskiego.Oryginalna treść jest dostępna na stackexchange, za co dziękujemy za licencję cc by-sa 3.0, w ramach której jest rozpowszechniana.
Loading...